Тёмный

Turkish Mathematical Olympiad | 2009 Q1 

Michael Penn
Подписаться 301 тыс.
Просмотров 58 тыс.
50% 1

We present a solution to a problem from the Turkish Mathematical Olympiad. This problem involves determining all prime numbers p such that a polynomial evaluated at p is the square of a positive integer.
Please Subscribe: ru-vid.com...
Merch: teespring.com/stores/michael-...
Personal Website: www.michael-penn.net
Randolph College Math: www.randolphcollege.edu/mathem...
Randolph College Math and Science on Facebook: / randolph.science
Research Gate profile: www.researchgate.net/profile/...
Google Scholar profile: scholar.google.com/citations?...
If you are going to use an ad-blocker, considering using brave and tipping me BAT!
brave.com/sdp793
Buy textbooks here and help me out: amzn.to/31Bj9ye
Buy an amazon gift card and help me out: amzn.to/2PComAf
Books I like:
Abstract Algebra:
Judson(online): abstract.ups.edu/
Judson(print): amzn.to/2Xg92wD
Dummit and Foote: amzn.to/2zYOrok
Gallian: amzn.to/2zg4YEo
Artin: amzn.to/2LQ8l7C
Differential Forms:
Bachman: amzn.to/2z9wljH
Number Theory:
Crisman(online): math.gordon.edu/ntic/
Strayer: amzn.to/3bXwLah
Andrews: amzn.to/2zWlOZ0
Analysis:
Abbot: amzn.to/3cwYtuF
How to think about Analysis: amzn.to/2AIhwVm
Calculus:
OpenStax(online): openstax.org/subjects/math
OpenStax Vol 1: amzn.to/2zlreN8
OpenStax Vol 2: amzn.to/2TtwoxH
OpenStax Vol 3: amzn.to/3bPJ3Bn
My Filming Equipment:
Camera: amzn.to/3kx2JzE
Lense: amzn.to/2PFxPXA
Audio Recorder: amzn.to/2XLzkaZ
Microphones: amzn.to/3fJED0T
Lights: amzn.to/2XHxRT0
White Chalk: amzn.to/3ipu3Oh
Color Chalk: amzn.to/2XL6eIJ

Опубликовано:

 

20 авг 2020

Поделиться:

Ссылка:

Скачать:

Готовим ссылку...

Добавить в:

Мой плейлист
Посмотреть позже
Комментарии : 215   
@PeperazziTube
@PeperazziTube 3 года назад
The funny thing happens if you brute-force in a programming language with insufficient decimal precision. As expected 2,7 and 11 are answers, but 19262791 also comes out. The reason: sqrt(p**3-4p+9) with p=19262791 is really close to an integer(84...x10**9 + 0.9999961...)
@TechToppers
@TechToppers 3 года назад
That's why human proofs are human proofs. No computer can battle us.
@ETiDeQuenVesSendo
@ETiDeQuenVesSendo 3 года назад
Why use decimals in a program?
@TechToppers
@TechToppers 3 года назад
@@ETiDeQuenVesSendo I don't think it was intentional. The computer might have approximated it.
@leif1075
@leif1075 3 года назад
But that's incorrect since it's not an integer
@rafael7696
@rafael7696 3 года назад
Hi Michael. I am professor in Spain. I enjoy very much all your imos and Putnam problems. I watch all them. Very, very good job. Thanks for your videos.
@siddeshpatnaik794
@siddeshpatnaik794 3 года назад
Your channel has made me love math again I hope that in the future you'll make videos on Fourier Transform because it's the only topic which is missing from your channel and there are so many interesting concepts about Fourier transform
@SimplyChrisRLP
@SimplyChrisRLP 3 года назад
I think the hardest part is always coming up with the hints giving the right direction. How do you identify what approach/hints to take?
@therealAQ
@therealAQ 3 года назад
might seem obscure, but i think knowing about elliptic curves would make the "hints" more obvious.
@HarmonicEpsilonDelta
@HarmonicEpsilonDelta 3 года назад
Experience and intuition
@lakshaygupta9061
@lakshaygupta9061 3 года назад
@@John-tb2ds stop spamming!
@myoung1445
@myoung1445 3 года назад
@@John-tb2ds bloody hell. You folks are clutching at straws aren't you
@John-tb2ds
@John-tb2ds 3 года назад
@@myoung1445 no i just want you to be saved
@SerkanSonel
@SerkanSonel 3 года назад
Here is a shortcut slightly from 8:45, aside from p={2,3}, taking the square of both sides of an inequality ((p^2-2p-9)/9) =< n, We will get the desired bound for p, by dividing p^3 as in the lecture, hence directly concluded that p=
@alperenkoken
@alperenkoken 3 года назад
I am very happy now,please make more videos about Turkish Mathematical Olympiads. Also you can try this one: For all x,y which are real numbers, (i) f(f(x²)+y+f(y))=x²+2f(y) (ii) x≤y --› f(x)≤f(y) Find the all functions which is f:R--›R (Turkish Mathematical Olympiad 2012 Second Round Day 1 Question 3)
@berkeunal5773
@berkeunal5773 3 года назад
:)
@ahmetbekiraydn8130
@ahmetbekiraydn8130 3 года назад
Bir matematikçi olarak mutlu oldum, Olympiad sorulari nereden bulabilirim, yardımcı olabilir misin?
@Notthatkindofdr
@Notthatkindofdr 3 года назад
Nice problem! The only function is the most obvious one, but my proof of this is a couple of pages long. :-)
@alperenkoken
@alperenkoken 3 года назад
@@Notthatkindofdr my proof is long to,I asked @Michael Penn for seeing another proof which is shorter than my one
@Notthatkindofdr
@Notthatkindofdr 3 года назад
@@alperenkoken I have found another solution which does not need condition (ii) in the problem, and I think is maybe simpler than my previous solution.
@Sharpgamingvideos
@Sharpgamingvideos 3 года назад
Never occurred to me to use divisors to set up bounds! Thank you for bestowing these techniques upon us for free :)
@sword7163
@sword7163 3 года назад
yeah that's super sneaky!
@elephantdinosaur2284
@elephantdinosaur2284 3 года назад
For everyone asking about comparing n against p^2/4 at 8:48 and the early bound of looking at cases p>11 at 1:53 the comparisons aren't required at all and can be proved without it. They are for simplifying the argument a little. The bound of p>11: For any prime p, n^2 = 9 (mod p) implies (n-3)(n+3) = 0 (mod p) hence p divides either n-3 or n+3. Dropping the assumption p>11 doesn't affect any of the later calculations. The comparison with p^2/4: Combining the bound (p^2-2p-9)/3 < n with the identity n^2 = p^3-4p+9 we obtain the inequality (p^2-2p-9)^2 < 9(p^3-4p+9). After expanding, simplifying and factorising we obtain p(p-2)(p^2-11p-36)
@venkateshbabu1504
@venkateshbabu1504 3 года назад
Always the last constant determine the roots and always less than last remainder theorem and it tells something about how the last constant is distributed along the function. With the highest power as one. 2 7 and 9+2.
@alexanderhayes3011
@alexanderhayes3011 2 года назад
if we set the equation to x^2 and rearrange we can factor it as p(p^2-4)=(x+3)(x-3). p is prime so must divide either (x+3) or (x-3). The difference between these two terms =6 therefore let ap =x+-6 and p^2-4= a^2p+-6a. Now gather all terms on the left-hand side: p^2-a^2p-4+-6a=0 and use the quadratic equation to solve for p: p=(a^2+-sqrt(a^4+16+-24a))/2. Now we look at the discriminant. To be a whole number a^4+16+-24a must be a perfect square but a^4 and 16 are both already perfect squares. This means that 24a must be the cross term and it must equal to 8a^2. Therefore a=3. We can now rewrite the equation as p=(9+-sqrt((9+-4)^2)/2 and this simplifies to p=(9+-9+-4)/2 =(18+-4)/2 or +-4/2 which means p=2, 7, 11 after we discount the answer of negative 2
@georgelaing2578
@georgelaing2578 2 года назад
If you look at the original expression mod 4, you get p(cubed) + 1 is congruent to zero or 1. This means that p = 2 (which checks out) or p is of the form 4k + 3. This reduces the work considerably.
@nikitakipriyanov7260
@nikitakipriyanov7260 3 года назад
I don't understand why we need to consider two cases as in 9:36 and further. I was able to skip that assumption that "was going out from nowhere" and solve more straighforward way. On that board we had already found that 3n≥p²-2p+3. Squaring that, we have 9n²≥p⁴-4p³-14p²+36p+81. But we search for p's where n²=p³-4p+9 (from the problem statement), so we just substitute that into this inequality and we get a polynomial inequality for p. It only looks quartic and hard, in fact is's easy to solve: the free term 81 cancels out, so after combining all the terms we have p⁴-13p³-14p²+72p≤0. Let's forget for a minute p is a prime, just let's factor it out and leave a cubic poly'al. It happens that 2 is a root of that cubic, so it factorizes to (p-2)(p²-11p-36)¸ with irrational roots, one of which is negative, anf the other is around 13.6. So we have all four roots or our quartic (around -2.6, 0, 2, around 13.6). Solving this inequality by intervals method we find useable intervals are [around -2.6, 0] and [2, around 13.6]. Now, remembering p was a prime, this translates into p≤13. Easy part, testing each of p = 2, 3, 5, 7, 11, 13 we have left with 2, 7 and 11, whose are the answer (corresponding n's are 3, 18, 36).
@goodplacetostop2973
@goodplacetostop2973 3 года назад
15:25
@integralboi2900
@integralboi2900 3 года назад
How?
@sumukhhegde6677
@sumukhhegde6677 3 года назад
🙌🙌🙌
@user-mt9ux2di6u
@user-mt9ux2di6u 3 года назад
Always so helpful!
@goodplacetostop2973
@goodplacetostop2973 3 года назад
Integarahl boi How? Magic, I guess 🤷‍♂️
@goodplacetostop2973
@goodplacetostop2973 3 года назад
איתן גרינזייד 👍
@yusufnar6454
@yusufnar6454 3 года назад
Greetings from Turkey
@electrovector7212
@electrovector7212 3 года назад
Cheers from Turkey! Good problem anyway. Turkey's questions are really good compared with most of the countries -especially the term 2005-2015 the brightest of Turkey National Team-. I hope you can offer more questions from Turkey:)
@crazyAngol
@crazyAngol 3 года назад
Another way to solve it: p^3-4p=(n-3)(n+3) either n+3 or n-3 must be divisible by p so p^2-4=a(n+-3)=a(pa+-3)=a^2+-3a. Using the quadratic equation p=(a^2+-sqrt(a^4+16+-24a)/2 however the square root term must also be a perfect square otherwise the answer cannot be an integer therefore 8a=+-24 a^2. Therefore a=+-3. Feeding this back into the original equation (unnecessary because we could have used the previous equation but it is what I did) we get two separate quadratic. Nothing else is possible p^2-9p-22=0 or p^2-9p+14=0 one solution is impossible (-2), the others are 2,7,11
@TheQEDRoom
@TheQEDRoom 3 года назад
Another possible answer is to prove that n is divisible by 3. To do this, transpose 9 to the LHS and factor the RHS. Now assume for a moment that p>3. This means that 3 does not divide p (since p is a prime) but 3 must divide at least one of (p-2) or (p+2). Now, this means that p^3-4p is divisible by 3, and so n^2 is also divisible by 3. But this means that 3|n. Now, let n=3m. We have m^2=(p)(p-2)(p+2)/9+1. Rearranging, we have (m+1)(m-1)=p(p+2)(p-2)/9. But since p is odd, so are (p-2) and (p+2). This means that (m-1)(m+1) are both odd.But this implies that gcf(m+1,m-1)=1. Notice also that the two factors differ by 2. This is an important hint Also, since p is odd prime, gcf(p,p-2)=1, gcf(p+2,p-2)=1, gcf(p,p-2)=1. This also implies that any of p-2 or p+2 is divisible by 9. Assume 9|(p-2). This means that the product of any two of the three factors p, (p+2), and (p-2)/9 must be either 2 more or 2 less than the third. We have six cases, all of which results in a quadratic equation with one variable. Then solve for p. Now assume 9|(p+2). This means the product of any two of the three factors p, (p-2) and (p+2)/9 must be 2 more or 2 less than the third. We have six cases, all of which results in a quadratic equation with one variable. Then solve for p. Now, p=2 will be a solution. Since we assume that p>3, what remains is to test at p=3.
@elephantdinosaur2284
@elephantdinosaur2284 3 года назад
I might have missed something but I think the part about checking 6 cases only works if you know each of the three factors p, (p+2), (p-2)/9 are prime numbers. For e.g. 7 x 11 x 15 = 33 x 35. Each of the numbers on the left are co-prime to each other but if you take the difference of multiplying any two of them together with the third you wouldn't get 2.
@TheQEDRoom
@TheQEDRoom 3 года назад
@@elephantdinosaur2284 Thanks for noticing the error. I completely missed that part. And also the part where all (p), (p-2) and (p+2) differ by just 2, so no need for quadratic equation in the 6 cases I mentioned. I think I have to make more analysis to correct the error. Thanks!
@Kapigala
@Kapigala 3 года назад
Problem proposition - From Polish edition of Championnat des Jeux Mathématiques et Logiques final (2003, Question 18): Find all natural n such that : 3^n+4^n+5^n...+(n+2)^n = (n+3)^n.
@TechToppers
@TechToppers 3 года назад
Nice
@MathswithMuneer
@MathswithMuneer 3 года назад
Respect from one math teacher to another math teacher
@AK-fr8pq
@AK-fr8pq 3 года назад
Great shirt 👍🤗
@mohammedelhouari6468
@mohammedelhouari6468 3 года назад
there should have a resolution more short and more clear. i m sure of that
@brightyan07
@brightyan07 3 года назад
You can check the comments below nylmoulin Rui of using mod (3).
@quickyummy8120
@quickyummy8120 3 года назад
I found another one from youtube ru-vid.com/video/%D0%B2%D0%B8%D0%B4%D0%B5%D0%BE-ctA6JF22ypI.html
@leefisher6366
@leefisher6366 3 года назад
Instantly thought that primes must end in 1, 3, 7 or 9; and putting those digits through the formula needs to end up with a units digit of 1, 4, 5, 6, 9 or 0. (ie not 2, 3, 7 or 8). 1 minus 4 plus 9 is 6 (3*3*3) = 7 - 2 + 9 is 4 (7*7*7) = 3 - 8 + 9 is 4 (9*9*9) = 9 - 6 + 9 is 2 - so that's the only thing I've ruled out. The prime can't end in 9. Now let's see what I should have been doing...
@mdjwy
@mdjwy 3 года назад
Let p^3-4p+9=k^2 -> (p-2)p(p+2)=(k-3)(k+3), the product of three consecutive numbers with the difference 3 should be described into the product of two numbers with a difference of 6. If p-2 or p+2 is 9m (m=0, 1) (p-2)p(p+2) = m*3p*(3p+/-6), so satisfies conditions. Of course, I should strictly prove that m bigger than 1 cannot satisfies the condition, but ...
@leif1075
@leif1075 3 года назад
Who in the world would even think of doing this?? Why not move the 9 to tbe other side and factor it as p(p-2)(p+2)= (n-3)(n+3) and then plug in values to see what works??
@mcwulf25
@mcwulf25 2 года назад
Eqn factorises as (p-2) p (p+2) = (m-3)(m+3) For p>2 we have three consecutive odd numbers so one must be a multiple of 3. I'm fact a multiple of 9 as RHS is a multiple of 9 if it's a multiple of 3. RHS can be rewritten 9(k-1)(k+1). k=3m. Obvs p can't be 9 so either (p-2)=9 or (p+2)=9. In fact both give solutions with three consecutive odd numbers (k-1),(k+1),9 and 9,)k-1),(k+1). p=7 or 11.
@ilnurkhamidullin6133
@ilnurkhamidullin6133 3 года назад
There is a lot ways to solve, but try to think of this that way: p^3-4p=k^2-9 p^2(p-4)=(k+3)(k-3) Then we can think 'bout which of multipliers from k+3 and k-3 devides by p. For one reason it can' t be only k-3, because then it devides by p^2 and there is unequalety k-3>=p^2. Analogicly it can't be only k+3, so both of k+3 and k-3 devides by p, so 6 devides by p. And then we check p=2 or 3 by hands. And get that only 2 reaches
@alainrogez8485
@alainrogez8485 3 года назад
You did a mistake p^2(p-4) is not p^3-4p, it is p^3-4p^2.
@user-eb6vc2gs9e
@user-eb6vc2gs9e 3 года назад
p^2(p-4) = p^3-4p^2
@user-pj9fm4zh4v
@user-pj9fm4zh4v 3 года назад
英語と数学の勉強が同時にできる一石二鳥
@santiagoarce5672
@santiagoarce5672 3 года назад
That's funny. I've done the same thing for Japanese and math before. I watched 鈴木貫太郎's channel quite a lot in the past.
@therealAQ
@therealAQ 3 года назад
but... what was the rank of the elliptic curve?
@MrPejotah
@MrPejotah 3 года назад
So when you get to pm^2 +- 6m = p^2 -4, why can't we solve as a quadratic in p, and try to find m values that make p prime? When I try it doesn't work btw, cause I end up with the sqrt of m^4 +- 24m + 16 which can't be factored into squares afaik. Thx
@jeffreyhellrung
@jeffreyhellrung 3 года назад
This was actually the direction I took and I found it to be a little more direct. Even though m^4±24m+16 can't be factored into polynomials in m, the result can still be a perfect square for specific values of m. The basic observation is that the square root of m^4±24m+16 can't be "too far" from m^2 for large enough m, but it still needs to be an integer; and, actually, you're pretty much restricted to checking when m^4±24m+16 = (m^2+2)^2 or (m^2+4)^2. Only m = 3 ends up working, which leads to the 3 solutions p in {2, 7, 11}.
@mustafaunal1834
@mustafaunal1834 3 года назад
Nice.
@adognamedsally
@adognamedsally 3 года назад
So when he got to P|+-6m-4, why did he say that, because P≧11, P divides half of +-6m-4? Does it have something to do with the -4?
@beanhwak
@beanhwak 3 года назад
Let = represent congruence , that is a = b mod p means a is congruent to b mod p, p is prime. 64 = 100 mod 3 then 8 = - 10 mod 3 but not 8 = 10 mod 3 100 = 9 mod 13 then 10 = -3 mod 13 but not 10 = 3 mod 13 144 = 64 mod 5 then 12 = - 8 mod 5 but not 12 = 8 mod 5 So n^2 = 9 (mod p) we only have (n = 3 mod p) OR n = -3 (mod p ) but not both. Many books also use + AND - instead of + OR - , wondering if this is trivial in the video. I frequently look for materials in your videos to train students for math contests. Thank you.
@mcwulf25
@mcwulf25 3 года назад
That's a hard one. As usual I lasted till about half way.
@user-sh1ce3yw9f
@user-sh1ce3yw9f 3 года назад
please answer this. you said on 5:03 that if it is not even it must divide the odd part of the right side but that doesn't hold for every divisor. For instance, 3 divides 21, therefore, it divides 19 + 2. Similarly, we could say that it should divide 19 something that is false.
@Hiltok
@Hiltok 3 года назад
He is referring to p|(±6m-4) p|(±6m-4) --> p|2(±3m-2) --> p|2 or p|(±3m-2) Trivially, 2 is only even prime giving p|2. So, if p is not even (p>2): p|(±3m-2) which is equivalent to p|(3m±2) Now, consider m=odd: (3m±2)=odd, hence p divides the odd part of (±6m-4)=±2(3m±2). Consider m=even: (3m±2)=even, hence p|(3m±2) --> p|2(1½m±1) --> p|(1½m±1). This process can be repeated until you have removed all factors of 2, leaving p dividing some odd number that is equal to (3m±2)/2ª. That is what he meant by p divides the odd part of (±6m-4). The reduction to (3m±2) is sufficient for this problem. No real need to consider cases for (1½m±1) or (¾m±½) etc.
@quickyummy8120
@quickyummy8120 3 года назад
@@Hiltok I found another one from youtube ru-vid.com/video/%D0%B2%D0%B8%D0%B4%D0%B5%D0%BE-ctA6JF22ypI.html
@ThePharphis
@ThePharphis 3 года назад
Huh, I'm wondering about the +/- portions being added together. Can we do that? I suppose it's because they're both "+/-" rather than one of them being "-/+" ? I was thinking they may partially (or completely) cancel, otherwise. Thanks
@toast_recon
@toast_recon 3 года назад
Not sure what part he did that, you'll have to help me out. But in general you just have to be careful with +/-. If two +/- signs are *linked*, as in the choice of one forces the choice of the other, then it would be find to combine in some way. The whole reason -/+ even exists is to be linked with a +/-. If you've made two independent choices of +/-, you probably shouldn't combine them into 1 equation and instead split into two cases. Otherwise you'd have to index them or something so you remember to split into 4 cases in the end.
@ThePharphis
@ThePharphis 3 года назад
@@toast_recon sorry I don't rmember the time-stamp but he went from +/-3m (or something similar to +/-6m But I think I wrapped my head around it.
@MyOneFiftiethOfADollar
@MyOneFiftiethOfADollar 2 года назад
The assumption early on that p>=11 seemed unmotivated. Hardest part of proving something difficult is to not use a truth that follows from already knowing the answer.
@wagsman9999
@wagsman9999 3 года назад
I enjoy watching these proofs, but they punctuate how weak I am this area (number theory, analysis). Can anyone recommend entry level book(s). (I have some math background - engineer, calculus tutor).
@destemanyagi
@destemanyagi 3 года назад
türkler için söylüyorum bu soru 2009 tübitak lise matematik olimpiyatları 2.aşama 1.gün 1.soru
@Balequalm
@Balequalm 3 года назад
Zannetmezdim ki Türkiye'den izleyen biri olsun sâhiden sfdfdf. Merâk ediyordum ikinci aşama mı birinci mi diye ama bir için olmaz gibi geliyordu. Sağ ol.
@Emre-yg3nn
@Emre-yg3nn 3 года назад
Hey maşallah
@Emre-yg3nn
@Emre-yg3nn 3 года назад
@Hellow valla ben açıyorum 5. dakikadan sonra kafam almıyor kopup kapatıyorum
@sirajyahya9049
@sirajyahya9049 3 года назад
Az Tr Az Tr sjbdhshdhd (beli burdada)
@KurstyCrabbb13
@KurstyCrabbb13 3 года назад
Baba her yerdeyiz
@rogueartist9419
@rogueartist9419 3 года назад
why did u take n≤p²/4... why not p²/5 or something else? we would have got a quadratic equation in both cases..
@jakobfredin4909
@jakobfredin4909 3 года назад
I'm wondering about this as well
@ArdiSatriawan
@ArdiSatriawan 3 года назад
Wondering this as well, it looks like black magic to me.
@rogueartist9419
@rogueartist9419 3 года назад
@Hellow so what ?
@jakobfredin4909
@jakobfredin4909 3 года назад
@Jakub Frei There has to be some other reason no? If that's it then we could just switch out 4 for any perfect square n?
@rumbaallday5575
@rumbaallday5575 3 года назад
Just as he said, it comes from nowhere... prolly tried with a·p² and found that with 1/4 both inequalities had a useful solution.
@phasm42
@phasm42 3 года назад
Surf Arrakis 😄 Miller's World would be another prime location.
@user-ue5gl5hh4c
@user-ue5gl5hh4c 3 года назад
How about your mega favorite number??
@karthikkrishnaswami3164
@karthikkrishnaswami3164 2 года назад
The technique provided gives us new insights, but the solution is way too complicated. Just consider 2, get it out of the way and for odd primes, consider primes to be 3k+/-1. For both cases lhs is 0 mod 3. This implies it is 0 mod 9 too as rhs is a perfect square. Hence p(p^2-4)=9n^2-9=9n(n^2-1) .case wise bashing-p+2=9,p-2=9 yields p=7,11. Hence p=2,7,11. Other cases don't yield solutions. Very easy :-)
@moonlightcocktail
@moonlightcocktail 3 года назад
I tried making the ff. Soln, and got the same answers as you, but I have no idea how to justify one of the steps I made...: Express the RHS as a DOTS. Then, we have (n-3)(n+3) = (p-2)(p)(p+2). Obsv: p even, then p 2, but RHS 0, so n 9. That's ok. So we need p odd. But p odd means n even. So we have the solutio: p = 2 Let n be 2g, then the lhs is ( 2g+3 ) (2g - 3), so p divides one of these terms, let one be Ap, the other is Ap + 6 or Ap - 6. We have either, Ap(Ap + 6) = (p-2)(p)(p+2). Ap (Ap - 6) = (p-2)(p)(p+2). Then transpose around to get A = ((p^2)-4)/(Ap + 6) or ((p^2)-4)/(Ap - 6) Treat the denominator and numerator as polynomials of p and use synthetic division, to get A = (1/A)p + (6/A^2)(p^0) + ((36/(A^2))-4)/(Ap+6) or A = (1/A)p + (6/A^2)(p^0) + ((36/(A^2))-4)/(Ap-6) Here is the step I have trouble with. There is a weak argument here, since A is an integer, so the term ((36/(A^2))-4)/(Ap-6) must be 0. I don't know how to connect the two but this is what solvds the problem, anyways if this term is nonzero we have an ugly-looking fraction for all three terms, and... I just don't know how to justify it. Even if the (1/A)p + (6/A^2)(p^0) terms were non integers, it's still more nice and likely to be that the sum of those two is an integer (?) Without the " interference" from the third term. But, if it is zero, then A = +3 or -3. It's not really important in this solution if we chose to use 2A + 6 or -6 earlier... isn't that strange? Now we solve the equation for p: The -3 solution: A = (1/A)p + (6/A^2) + ((36/(A^2))-4)/(Ap-6) -3 = -p/3 + 2/3; 9+2 = 11 = p. The 3 solution: 3 = p/3 + 2/3; 9-2 = 7 =p In addition to the 2 from earlier we now have all solutions of p.
@ireallydontknow3299
@ireallydontknow3299 3 года назад
for the part you have trouble with: If |A| > 6 (possibly there's a bound lower than this that works, but this was the first one I tried), then |1/Ap| < 1 |6/A^2| < 1 |(36/A^2 - 4)/(Ap +- 6)| |RHS| = |LHS| = |A| > 6, hence 3 > 6, absurd. And then you check what happens for |A|
@danibarack552
@danibarack552 3 года назад
"notice how i've changed the sign but that's kind of ok as well" Yet he had to explain why p divides something written as p*(...). If you're gonna explain all the obvious parts then explain the not so obvious ones as well
@jonathanjacobson7012
@jonathanjacobson7012 3 года назад
I reached the equation (p-2)*p*(p+2)=(n-3)*(n+3), which means that p divides either (n-3) or (n+3), but couldn't proceed any further without your help. Thanks!
@AlephThree
@AlephThree 3 года назад
I went here at first as well, but since p-2 and p+2 aren’t necessarily prime I couldn’t see how to go any further.
@mcwulf25
@mcwulf25 2 года назад
@@AlephThree They aren't prime. One must be a multiple of 3 in fact because there are 3 consecutive odd numbers there. Which means 3 divides the RHS. Which means 9 divides the RHS as 3 divides both n-3 and n+3. So we know p-2=9 or p+2=9. These give our solutions.
@siddharthavlash1982
@siddharthavlash1982 Год назад
if we set the equation to x^2 and rearrange we can factor it as p(p^2-4)=(x+3)(x-3). p is prime so must divide either (x+3) or (x-3). The difference between these two terms =6 therefore let ap =x+-6 and p^2-4= a^2p+-6a. Now gather all terms on the left-hand side: p^2-a^2p-4+-6a=0 and use the quadratic equation to solve for p: p=(a^2+-sqrt(a^4+16+-24a))/2. Now we look at the discriminant. To be a whole number a^4+16+-24a must be a perfect square but a^4 and 16 are both already perfect squares. This means that 24a must be the cross term and it must equal to 8a^2. Therefore a=3. We can now rewrite the equation as p=(9+-sqrt((9+-4)^2)/2 and this simplifies to p=(9+-9+-4)/2 =(18+-4)/2 or +-4/2 which means p=2, 7, 11 after we discount the answer of negative 2.
@soufianeaitabbou3727
@soufianeaitabbou3727 2 года назад
CAN SOMEONE TELL ME WHAT WE NOTICED THEN WE CHOOSED P^2/4 AS A BOUND FOR N IT STILL SEEMS ABSURD FOR ME
@jadegrace1312
@jadegrace1312 3 года назад
Unless I'm missing something, saying p|(3m+2) means p≤3m+2, isn't valid because of negative numbers. For example, p=13 and 3m+2=-52. Obviously this particular m and p doesn't satisfy the equation, but it still is a counter example to p|(3m+2)→p≤3m+2
@noumaneelgaou1624
@noumaneelgaou1624 3 года назад
I m morrocan students and Next years l will study in first years in university can participate international mathematical competition IMC ? if I can how to inscrire in this competition
@JoseFernandes-js7ep
@JoseFernandes-js7ep 3 года назад
You have to participate in the national math Olympics first. But in my time the IMO werw for pre-universitary students.
@ThePharphis
@ThePharphis 3 года назад
Can someone explain the simplification with the signs at 5:14? I can't quite follow that
@hybmnzz2658
@hybmnzz2658 3 года назад
+- operator is a built in OR statement. p | 6m-4 -> p | 2(3m-2) -> p | 3m-2 p | -6m-4 -> p | -2(3m+2) -> p | 3m+2 So in one statement p | 3m +- 2
@ThePharphis
@ThePharphis 3 года назад
@@hybmnzz2658 Ok that's much clearer! Thanks
@destemanyagi
@destemanyagi 3 года назад
nerdesiniz beyler
@usuyus
@usuyus 3 года назад
!!
@AlperNahir
@AlperNahir 3 года назад
🇹🇷
@huseyinturgut5026
@huseyinturgut5026 3 года назад
I am here
@egillandersson1780
@egillandersson1780 3 года назад
Oh ! Yet another transatlantic misunderstanding ! I read Q instead of 9 : a bit more complicated, isn't it 😅
@mcwulf25
@mcwulf25 2 года назад
I would never be able to do that. Not with that method anyway. Maybe there's another....?
@alboris8203
@alboris8203 2 года назад
Subbing for the Dune shirt lol
@arandomcube3540
@arandomcube3540 3 года назад
2:12 => assumes p >= 11 14:50 => gives all prime numbers under 13 as solutions to try, and then so 2 and 7 are. Excuse me? Where did I get this wrong please
@myxail0
@myxail0 3 года назад
well, assuming p>=11, u have 11 and 13. So u need to check them. But you still need to check primes under 11
@arandomcube3540
@arandomcube3540 3 года назад
@@myxail0 (I understood it juste after sending the comment 😂 but thanks 👍)
@dmitryweinstein315
@dmitryweinstein315 3 года назад
It’s a solution but not a very elegant one; could it be that the setters in the olympiad had in mind something simpler? E.g. transform the equation into (n-3)(n+3)=p(p-2)(p+2), or, because obviously both parts are divisible by 9, into 9(m-1)(m+1)=p(p-2)(p+2), and then prove that either p=2, m=1, or both sides represent a product of three consecutive odd numbers, one of which is 9, i.e. p=7 or p=11.
@swenji9113
@swenji9113 3 года назад
But why wouldn't it be the product of three odd numbers, one of which is A MULTIPLE of 9, rather than simply 9?
@dmitryweinstein315
@dmitryweinstein315 3 года назад
@@swenji9113 because the left part should be the product of the same three consecutive odd numbers as the right part (I don’t think it can work otherwise). Two of those numbers are m-1 and m+1 and the third one is 9.
@dmitryweinstein315
@dmitryweinstein315 3 года назад
@@swenji9113 @@swenji9113 because the left part should be the product of the same three consecutive odd numbers as the right part (I don’t think it can work otherwise, which I think should be easy to prove). Two of those numbers are m-1 and m+1 and the third one is 9.
@donaldbiden7927
@donaldbiden7927 3 года назад
RU-vid has changed
@OlegMarchuk
@OlegMarchuk 16 дней назад
It's 9! Wow. I thought what is this q means :)
@brucestevenson2933
@brucestevenson2933 3 года назад
2:20 Why he can take square on both sides? (n^2 ≡ 9 (mod p) ⇒ n ≡ ±3 (mod p)). Is this a theorem?
@pollitojb3731
@pollitojb3731 3 года назад
Maybe n^2 = 9 [p] n^2 - 9 = 0 [p] n^2 - 3^2 =0 [p] (n+3)(n-3) = 0 [p] So p|n+3 and p|n-3 Meaning n+3 = 0 [p] and n-3 = 0 [p] Or n = +-3 [p]
@brucestevenson2933
@brucestevenson2933 3 года назад
@@pollitojb3731 Got it! Thanks a lot!
@fahad-xi-a8260
@fahad-xi-a8260 3 года назад
@@pollitojb3731 for p must be greater than 3
@TechToppers
@TechToppers 3 года назад
Yes it's a theorem... P should be prime but...
@blackpenredpen2797
@blackpenredpen2797 3 года назад
I got three different prime number which satisfied the conditions. Those primes are 3,7 and 11. I got these answer through brute force 😅😅
@blackpenredpen2797
@blackpenredpen2797 3 года назад
I meant 2,7, and 11😅😅
@sergiokorochinsky49
@sergiokorochinsky49 3 года назад
5:12 5:17 ...wow, I've never seen that in my life. Still not convinced it is true.
@angelfranciscotorresnovero4168
@angelfranciscotorresnovero4168 3 года назад
If a|b*c and gcd(a,b)=1. Then a|c. Demonstration: Since a|b*c then there is an integer k such that b*c=a*k. Also, because gcd(a,b)=1 then there are integers u,v such that au + bv =1. Multiplying both sides of the last ecuation by c we get that acu + bcv = c. Grouping terms, we get a(cu) + (bc)v = c. But because b*c = ak, then we have a(cu) + (ak)v = c, and factorizing a from the left handside of the ecuation we get a(cu + kv) = c. This implies that a|c because there exists an integer m (where m= cu+kv) such that c=am.
@angelfranciscotorresnovero4168
@angelfranciscotorresnovero4168 3 года назад
Michael used this property, with a =p, b= 2, and c=3m +-2
@mehmetboraginali1712
@mehmetboraginali1712 3 года назад
Valla helal
@blankino-1824
@blankino-1824 3 года назад
Why could we make the assumption that n is bigger than or equal to 11?
@Pacuvio25
@Pacuvio25 3 года назад
It is the first prime after 9.
@blankino-1824
@blankino-1824 3 года назад
@@Pacuvio25 so? Sorry, I still don't quite get it
@demenion3521
@demenion3521 3 года назад
he just made this assumption to make the modulo part a bit easier. but in the end he checked all the smaller cases anyway
@iabervon
@iabervon 3 года назад
He's trying to find some M such that p < M for all solutions, and he doesn't care about M being the least upper bound. He can just pick M to be at least 12. Then, if p 11. That's where he finds he can prove it for M = 16.
@paoloboldrini7540
@paoloboldrini7540 3 года назад
think i solved in a similar but (for me) easier way. After reasoning mod p we can say p|a+3 or p|a-3. We also know for FLT p³=p (mod 3) so 3|p³-4p+9=a² and 3|a because 3 it's prime. I write the first case (p|a+3), the second is similar. a²=k²p²-6kp+9=p³-4p+9 so p²-k²p+(6k-4)=0 delta=k^4-24k+16, p is an integer so delta must be a perfect square. It's easy to see that delta=(k²-1)²+2k²-24k+15. Because it's a square it can't be (k²-2)
@aks7451
@aks7451 3 года назад
Can you solve this problem? Find all primes of the form p^p+2 such that p is also a prime. I’ve asked lots of people and none of them could help.
@doctorb9264
@doctorb9264 3 года назад
interesting problem for sure.
@ancientwisdom7993
@ancientwisdom7993 3 года назад
This looks great - I suggest u post this problem when Michael posts his daily video which he uploads around 5:30 PM india time - most interested viewers usually check his latest video. I have seen a somewhat similar problem solved recently
@qwe-oq2un
@qwe-oq2un 2 года назад
Very hard
@qwe-oq2un
@qwe-oq2un 2 года назад
Very very very
@JM-us3fr
@JM-us3fr 3 года назад
Okay, and that's a good place to stop.... ...you know.... at the end...
@yuvrajsarda6660
@yuvrajsarda6660 3 года назад
That was quite long....
@MizardXYT
@MizardXYT 3 года назад
Not a prime, but 646³ - 4∙646 + 9 = 16419²
@paoloboldrini7540
@paoloboldrini7540 3 года назад
how have you found? ahahahah
@giuseppebassi7406
@giuseppebassi7406 3 года назад
Definetly a really hard problem, I tried this, couldn't solve it and then I watched the solution: impossible!
@klementhajrullaj1222
@klementhajrullaj1222 2 года назад
Your hairs in every video, are even like this, like that! 😀😉 ...
@xaxuser5033
@xaxuser5033 3 года назад
plzz try this cool problem : find all functions that satisfy the following condition , for all x,y real numbers , f(x^2)-f(y^2)《(f(x)+y)(x-f(y))
@williamadams137
@williamadams137 3 года назад
XaXuser What does the symbol “《 “ mean?
@xaxuser5033
@xaxuser5033 3 года назад
@@williamadams137 less than or equal
@user-vg1qo5gi3l
@user-vg1qo5gi3l 3 года назад
@@williamadams137 Maybe less or equal?
@stormhoof
@stormhoof 3 года назад
Try his main approaches. x=0, y=a x=a, y=0 x=a, y=a x=a, y=-a Reply in the comments with any significant findings from those four approaches & then let’s work from there.
@ivangonzalezserra8121
@ivangonzalezserra8121 3 года назад
The solution is f(x)=x and f(x)=-x, am I wrong???
@osnapitzkaan
@osnapitzkaan 3 года назад
🇹🇷🇹🇷🇹🇷🇹🇷🇹🇷
@sumukhhegde6677
@sumukhhegde6677 3 года назад
🖕
@elie.makdissi
@elie.makdissi 3 года назад
🤫🖕
@________6295
@________6295 3 года назад
@@sumukhhegde6677 why did u do that?
@mahmuttuncer9978
@mahmuttuncer9978 3 года назад
:=)
@elie.makdissi
@elie.makdissi 3 года назад
@@________6295 🤫🤦‍♂️
@yeetyeet7312
@yeetyeet7312 3 года назад
Noice
@stepone3040
@stepone3040 3 года назад
😩😩😩
@gatocomcirrose
@gatocomcirrose 3 года назад
can you please try this question: find all natural numbers a, b and c such that a+2020=b+c 2020a+1=bc *if someone here in the comments know how to do pls help me too :)
@integralboi2900
@integralboi2900 3 года назад
I suggest looking at the general case by replacing 2020 by an integer, n.
@aadityajha7502
@aadityajha7502 3 года назад
Let there be a quadratic equation with roots b and c. Put values of b+c and bc into it in terms of a, Then calculate discriminant part and solve values of a for which discriminant is perfect square. Once a is known, find b and c using that quadratic equation. This can be generalized to any n
@sumukhhegde6677
@sumukhhegde6677 3 года назад
We get b=2020.111 .. not a natural number.. so no solution
@suyashmisra7406
@suyashmisra7406 3 года назад
I've edited this comment because i made a calculation mistake ;-; I believe a solution exists. you can use AM GM inequality to show that (a+2020)/2>=sq root(2020*a+1). sqaure both sides and you have a quadratic equation,the detereminant of which is sq(2020)-4*(sq(2020)-1) which is 16,thus a real a exists satisfying these relations. as many others have already pointed out,forming a quadratic with b and c as roots is the most intuitive way to go about this.
@gatocomcirrose
@gatocomcirrose 3 года назад
@@suyashmisra7406 from where that +1 inside of the square come from if you're doing the inequality with a and 2020? shouldn't be (a+2020)/2≥sqrt(2020a) ?
@mehmetburakgonul4102
@mehmetburakgonul4102 3 года назад
🇹🇷🇹🇷🇹🇷🇹🇷
@waffleonquaffle
@waffleonquaffle 3 года назад
No offense, but if that's how math is taught in the US, I can see how people hate it
Далее
Brazilian Math Olympiad | 2018 Q15
11:46
Просмотров 31 тыс.
A team selection number theory problem.
13:41
Просмотров 46 тыс.
2DROTS vs RISENHAHA! КУБОК ФИФЕРОВ 2 ТУР
11:31
МОЙ НОВЫЙ ДОМ
1:01:04
Просмотров 2,1 млн
Greek Mathematics Olympiad | 2008 Q2
13:01
Просмотров 45 тыс.
Irish Mathematical Olympiad | 2009 Q3
12:47
Просмотров 51 тыс.
find the integer solutions of this cubic
15:29
Просмотров 24 тыс.
The smallest such prime...
16:44
Просмотров 56 тыс.
9 tips to help you PROVE MATH THEOREMS
15:07
Просмотров 107 тыс.
A Diophantine Equation from Turkish Math Olympiads
8:52
INCORRECT PROOF of Fermat's Last Theorem
10:20
Просмотров 46 тыс.
New Zealand Mathematical Olympiad 2019 Question 5
18:42
Swedish Mathematics Olympiad | 2002 Question 4
14:19
Просмотров 310 тыс.
2DROTS vs RISENHAHA! КУБОК ФИФЕРОВ 2 ТУР
11:31